help i need perimeter of these

Help I Need Perimeter Of These

Answers

Answer 1

Answer:  A= 52cm  B= 94mm  C= 9m  D= 17cm

Step-by-step explanation:

A) You first need to find the missing sides by doing

11-9= 2

and 15-9= 6

you then add it all together

6+2+9+9+15+11=52cm

B) You first need to find the missing sides by doing

25-18=7

and 22-6= 16

you then add it all together

25+22+18+16+7+6=94mm

C) You first need to find the missing sides by doing

0.5+1=1.5 then 2.5-1.5=1

and 1.2+0.8=2 then 2-0.5=1.5

you then add the numbers all together

1+1.5+0.8+1+0.5+0.5+1.2+2.5=9m

D) You first need to find the missing sides by doing

2.5+1= 3.5 then 3.5-2=1.5

and 1.5+2.5=4 then 5-4=1

you then add all the numbers together

2.5+2.5+1.5+1+5+2+1+1.5=17cm


Related Questions


On Thursday the Meat King Market sold 210 pounds of ground beef. On Friday they sold twice
that amount. On Saturday they only sold 130 pounds. How much more meat did they sell on
Friday than Saturday?

Answers

Answer:

290

Step-by-step explanation:

210 x 2 = 420

420 - 130 = 290

Which expression is equivalent to the expression below?
7p+3(2q+9p)

a.7(p+6q+27p)7(p+6q+27p)
b.-2p+12q−2p+12q
c.-p+9q−p+9q
d.34p+6q34p+6q

Answers

The equivalent expression will be;

⇒ 34p + 6q

What is an expression?

Mathematical expression is defined as the collection of the numbers variables and functions by using operations like addition, subtraction, multiplication, and division.

Given that;

The expression is,

⇒ 7p + 3 (2q + 9p)

Now,

Since, The expression is,

⇒ 7p + 3 (2q + 9p)

Hence, Solve for equivalent expression as;

⇒ 7p + 3 (2q + 9p)

⇒ 7p + 6q + 27p

⇒ 34p + 6q

Thus, The equivalent expression is find as,

⇒ 34p + 6q

Learn more about the mathematical expression visit:

brainly.com/question/1859113

#SPJ1

Write Expressions Word Problems 1.
You run a small business and would like to print some business cards.
Staples offers color business cards for a small fee.
The cost is $.75 per card.
A) Write an expression for the total cost (in dollars) if you print b business cards. B) How much does it cost you to print 50 business cards?

Answers

Step-by-step explanation:

cost of business card - $.75 ( per card )

I want to print 50

So we will multiply 75 × 50

75

×50

00

355×

=375

$.375 will be the cost of the same

The ratio of females to males at a movie is 8 7 If there are 28 males how many females
are in the movie

Answers

Answer: 32 females

Step-by-step explanation:

28 divided by 7 = 4

so, multiply 8 by 4 as well.

Answer: 32 females are at the movie

Step-by-step explanation:

8(females):7(males)

x:28

28/7= 4

x=4x8

x=32

There are 32 females

The point is located on the x-axis and 10 units to the left of the y-axis

Answers

The ordered pair is located at the x-y coordinate at (- 10, 0).

What is an ordered pair?

An ordered pair is made up of the ordinate and the abscissa of the x coordinate, with two values given in parenthesis in a certain sequence.

Pair in Order = (x, y)

x is the abscissa, the distance measure of a point from the primary axis x

y is the ordinate, the distance measure of a point from the secondary axis y

Given, A point is located on the x-axis which is y = 0, and 10 units to the left of the y-axis which is x = - 10.

So, The location of the ordered pair (x, y) is (- 10, 0).

learn more about ordered pairs here :

https://brainly.com/question/28874341

#SPJ1

How many 3/4-cup servings are in 5/12 cups of yogurt?

Answers

there are 8/9 cups of yogur

A restaurant manager can spend at most $600 a day for operating costs and payroll. It costs $100 each day to operate the bank and $50 a day for each employee. Use the following inequality to determine how many employees the manager can afford for the day, at most:

50x + 100 ≤ 600

Answers

10 employees a day I believe

An object is attached to a spring that is stretched and released. The equation d= -8cos (pi/6 t) the distance, d, of the object in inches above or below the rest position as a function of time, t, in seconds. Approximately when will the object be 6 inches above the rest position? Round to the nearest hundredth, if necessary.

Answers

The time at which the object is 6 inches above the rest position will be 4.62 seconds.

What is a function?

A function is an assertion, concept, or principle that establishes an association between two variables. Functions may be found throughout mathematics and are essential for the development of significant links.

An item is connected to a spring that is extended and delivered. The condition d= - 8cos [(π/6) t] the distance, d, of the item creeps above or underneath the rest position as an element of time, t, in a flash.

d= - 8cos [(π/6) t]

If d = 6, then the value of the variable 't' is calculated as,

6 = - 8cos [(π/6) t]

- 3/4 = cos [(π/6) t]

cos [(77/100)π] = cos [(π/6) t]

Compare the angle, then we have

(π/6) t = (77/100)π

t / 6 = 77 / 100

t = 4.62 seconds

The time at which the item is 6 crawls over the rest position will be 4.62 seconds.

More about the function link is given below.

https://brainly.com/question/5245372

#SPJ1

Which method is best for root finding?

Answers

Answer:

on the value of the root may produce a value of the polynomial at the approximate root that is of the order of. For avoiding these problems, methods have been elaborated, which compute all roots simultaneously, to any desired accuracy. Presently the most efficient method is Aberth method.

Please help me im trying to finish before deadline

Answers

The theorem that proves <DAE=<BAC is Pythagoras theorem.

What is  Pythagoras theorem ?The Pythagorean Theorem, often known as Pythagoras Theorem, is a crucial concept in mathematics that describes how the sides of a right-angled triangle relate to one another. Pythagorean triples are another name for the sides of the right triangle. Here, examples help to demonstrate the formula and proof of this theorem.In essence, the Pythagorean theorem is used to determine a triangle's angle and length of an unknown side. This theorem allows us to obtain the hypotenuse, perpendicular, and base formulas. In a right-angled triangle, the square of the hypotenuse side is equal to the sum of the squares of the other two sides, according to Pythagoras's Theorem.

To learn more about Pythagoras theorem refer to:

https://brainly.com/question/343682

#SPJ1

how long will it take (in years) for the disintegration rate of this sample to drop to 809 disintegrations per minute?

Answers

Consequently, it would take roughly 690,64 years for the disintegrations to disappear.

What is disintegrations?

Disintegration occurs when a whole entity breaks up into smaller pieces or simply vanishes. Disintegration is the breakdown, fragmentation, or spontaneous disintegration of something. Knowing that integration brings things or people together will help you to understand that disintegration refers to things breaking apart.

Here,

The formula for this is Activity = decay constant x atoms.

1238 = 2.64 x 1010 times the decay constant

decay constant is equal to 4.76 x 10-8 min-1.

Currently, the disintegrations are being slowed to 809 disintegrations per minute.

Ln(809) = Ln(2.64 x 1010) - Ln(4.76 x 10-8 x t)

t=3.63x108min = 690.64y

Consequently, it would take roughly 690,64 years for the disintegrations to disappear.

To know more about disintegrations ,visit

https://brainly.com/question/7140420

#SPJ4

Which of the following equations is parallel to 8x - 10y = -2 and contains (1, -2)?
1. 4x - 5y = -2
2. 5x - 4y = 6
3. 4x - 5y = 6
4. 4x - 5y = 14


needed as soon as possible

Answers

The linear equation that is parallel to 8x - 10y = -2 and contains (1, -2) is:

4x - 5y = 14

So the last option is the correct one.

Which of the following equations is parallel to 8x - 10y = -2?

Remember that two linear equations are parallel only if both lines have the same slope (or coefficients if it is written in standard form) and different y-intercept.

Here our line is:

8x - 10y = -2

If we divide all the equation by 2, we will get:

(8x - 10y)/2 = -2/2

4x - 5y = -1

So any equation of the form:

4x - 5y = c, where c ≠ -1

Is parallel to 8x - 10y = -2

Now we want our line to pass through (1, -2), replacing these values in our line we get:

4x - 5y = c

4*1 -  5*-2 = c

4 + 10 = c

14 = c

Then the linear equation is:

4x - 5y = 14

So the correct option is the last one.

Learn more about linear equations:

https://brainly.com/question/1884491

#SPJ1

Lin says she has memorized the lengths of a few right triangles, for example, 3, 4, and 5. She is trying to compile a list of several right triangles but needs your help.

Select all the lengths of triangles that are right.

1.) 8, 9, 64

2.) 2,4,6

3.) 1, 1, V2

4.) 7,19,22

5.) 5, 12, 13

Answers

Number 5 it is hope that helps

Find the equation of a parabola with its vertex at the origin and directrix x = 3.

Answers

The equation of a parabola is y²+12x=0.

In mathematics, what is a parabola?

A parabola is a U-shaped plane curve in which any point is an equal distance from both a fixed point (known as the focus) and a fixed straight line (known as the directrix).

What exactly are the three parabola equations?

Parabola functions are classified into three types: standard form, vertex form, and intercept form (also known as factored).

as per the given question:

We have a vertex at (0,0) and the directrix is x = 3, a line parallel to the y-axis, it must have a focus at (3,0).

The parabola equation represents the locus of a point (x, y) that moves so that its distance from x=3 and (3,0) is equal. As a result, the parabola equation is, An equation of the parabola represents the locus of a point (x, y), which moves so that its distance from x=3 and (−3,0) are equal. Hence, the equation of a parabola is

(x−(−3))² + (y−0)² = (x−3)²

or (x+3)² + y² = (x−3)²

or x² + 6x + 9 + y² = x² − 6x + 9

or y² + 12x =0

Hence, The equation of a parabola is y²+12x=0.

To know more about the equation of parabola visit:

https://brainly.com/question/11911877

#SPJ1

A box in the shape of a rectangular prism has the dimensions shown. What is the length of the interior diagonal of the box? Round to the nearest tenth. Enter your answer in the box. A rectangular prism 60 centimeters wide, 80 centimeters long, and 100 centimeters tall. A diagonal d is drawn from the upper right rear corner to the lowest left front corner.

Answers

It is to be noted that the length of the interior diagonal of the box whose dimensions are Height 100cm, Lenght, 80 cm, and width 60 cm, is 140.4cm.

What is the formula for the length of a right rectangular prism?

Recall that the formula for the diagonal length of a right rectangular prism is calculated as √(l² + w² + h²) units. So, the formula for the length of a right rectangular prism's diagonal is √(l²+w²+h²), where l is the length, b is the breadth, and h is the height.

Plugging in the values we have:

d² = 60² + 80² + 100²

Solving for d, we get:

d = √(660² + 80² + 100²)

= √(3600 + 6400 + 10000)

= √(20000)

= 140.4 cm

Thus, the length of the interior diagonal of the box is 140.4 cm.

Learn more about interior diagonal:
https://brainly.com/question/12947624
#SPJ1

9. A straight line passes through points (2, 12) and (3, 8). What is the equation of the
line?

Answers

The equation is y = -4x + 20

A hook in an office storage closet can hold no more than 6 pounds. An order of jumbo paperclips weighs 2 pounds and an order of packing tape weighs 3 pounds. If x is the number of orders of paperclips and y is the number of orders of packing tape, which graph represents the overall equation represented by this scenario (all points may not apply to the scenario)?

On a coordinate plane, a solid straight line has a negative slope and goes through (0, 2) and (3, 0). Everything below and to the left of the line is shaded.

On a coordinate plane, a solid straight line has a negative slope and goes through (0, 2) and (3, 0). Everything above and to the right of the line is shaded.

On a coordinate plane, a solid straight line has a negative slope and goes through (0, 3) and (2, 0). Everything to the right of the line is shaded.

On a coordinate plane, a solid straight line has a negative slope and goes through (0, 3) and (2, 0). Everything to the left of the line is shaded.

Answers

The first graph shows how many of each order could fit into a bag that was hanging from a hook. First, a detailed explanation: * Let's discuss the issue's solution.

Find graph?

The quantity of orders of paperclips is x, and the weight of the order of jumbo paperclips is 2 pounds. The order of paperclips weighs 2 x = 2 x, the order of packing tape weighs 3 lbs., and the quantity of orders of packing tape is y. The weight of the tape order is 3 y = 3 y, the weight of the entire order is 2 x + 3 y, and the hook can only support a maximum weight of 6 y. 2x + 3y ≤ 6

The graph that depicts this inequality, Any line's equation is y = mx + c, where m represents its slope.

The y-intercept indicates to replace x in the equation by 0; the x-intercept indicates to replace y in the equation by 0; 2x - 3y 6 is the inequality. 2x + 3y = 6 is the equation for the line.

- Subtract 2x from both sides: 3y = 6 - 2x; - Divide both sides by 3: y = 6/3 - 2/3 x; - Locate the y-intercept: x = 0; y = 2;

The line crosses the y-axis at position (0 , 2)

– Locate the x-intercept.

∵ y = 0

∴ 0 = 2 - 2/3 x

- Adjust both sides by 2/3x.

∴ 2/3 x = 2

Increase both sides by 3.

Divide all sides by 2 to get the answer.

∴ x = 3

The line crosses the x-axis at position (3 , 0)

First and second from the figure.

To learn more about Graph refer to:

https://brainly.com/question/19040584

#SPJ1

Could someone pls explain on how to find the 5 number summary, Recall that the 5-number summary is the minimum, lower quartile, median, upper quartile, and maximum.


The numbers are 1 1 1 1 3 3 3 4 4 4 5 5 5 5 6 6 6 13 13 16 16 16 21 22 24 28 28

Answers

Maximum = 28, Minimum = 1, Lower Quartile = 3, Median = 5, and Upper Quartile = 16

The Five Number Summary is what?

The minimum, lower quartile (Q1), median (Q2), upper quartile (Q3), and maximum make up the five-number summary. The median is the middle value; Q1 is the median of the first half of the data, and Q3 is the median of the second half. The lowest is the smallest number, the maximum is the highest, the median is the intermediate value.

The 5 digit summary definition

minimal is a smaller number.

lower quartile: First-half median of the data.

median

upper quartile :Median of the second half of data

greatest number is maximum

The numbers are 1 1 1 1 3 3 3 4 4 4 5 5 5 5 6 6 6 13 13 16 16 16 21 22 24 28 28

minimum = smallest number = 1

lower quartile = Median of the first half of data = 1 1 1 1 3 3 3 4 4 4 5 5 5 = 3

median = median( middle value) = 5

upper quartile = Median of the second half of data = 6 6 6 13 13 16 16 16 21 22 24 28 28 = 16

maximum = largest number = 28

To know more about 5 number summary click the link:

brainly.com/question/29107036

#SPJ1

Booker T. Washington Middle School found two companies that would deliver healthy snacks to school. The tables below show the price charged by each company. How many more times does Company B charge per snack than Company A?

Answers

Two more times company B charge per snack than company A

What is meant by ratio?

A ratio in mathematics displays the multiplicative relationship between two numbers. In a bowl of fruit, for instance, the proportion of oranges to lemons is eight to six if there are eight oranges and six lemons (that is, 8:6, which is equivalent to the ratio 4:3). In a similar vein, the proportion of oranges to the overall amount of fruit is 8:14, and that of lemons to oranges is 6:8 (or 3:4). (or 4:7).

A ratio can have any number of numbers as its components, including counts of people or things, weights, lengths, and times, among other values.

From the two tables,

0.5/0.25=2

1/0.5=2

1.5/0.75=2

2/1=2

Therefore, 2 more times company B charge per snack than company A

To know more about ratio, visit:

https://brainly.com/question/13419413

#SPJ1

please help with this ​

Answers

The second price means how much the chips cost per gram. That's what the "/g" part means. Divided by gram. So, the second price of 1000 grams of Nameless chips gets you more chip per cent, because the bag charges $0.002 per gram instead of $0.003 per gram as the other does.

Step by step !!!!pease help

Answers

The option C is correct

What is the mathematical ratio?

Definitions: When the second number in the ordered pair, b, is not equal to 0, the ratio is expressed as a/b. An equation in which two ratios are made equal is known as a percentage. For instance, you could express the ratio as 1: 3 if there is 1 guy and 3 girls 

Given that:

Everglades Elementary school Purchases 125 fourth grade math's textbooks and 191 fifth grade math's textbooks

Total textbooks = 125 + 191 = 316 books

A box can contain maximum = 28 books

so the ratio of books and box will be = 316/28 = 11 box and 8/28

So total box will be 11 completely filled and the last one will be filled by 8/28

Learn more about ratio

https://brainly.com/question/12024093

#SPJ1

The school's math team has 14 new members this year, which makes a total of 34. The number of new members represents about what percent of the total membership?

Answers

41% of the math team's total members are brand-new recruits.

What is percentage ?

It is the ratio of two integers expressed as a decimal fraction of a whole number. It is a statistic for contrasting two sets of data, and the % sign is used to describe it as a percentage.

Percentages are often and creatively used. For instance, a lot of data is published as percentages in the media, including bank interest rates, retail discounts, and inflation rates. Percentages are essential for grasping the financial aspects of daily living.

The percentage of the entire membership that the number of new members represents is determined to be

=(14/34)×100

=41 %

0.41 x 100 = 41%

This means that 41% of the club is new members!

Thus, the number of new members is 41% of the total membership of the maths team.

Learn more about the percentage here:

brainly.com/question/26352729

#SPJ1

Write the linear equation in slope intercept form

Answers

[tex]y= - \frac{1}{3} z +2[/tex] is slope intercept form of given equation.

How do you write slope intercept form?Y = mx+b, where m is the slope and b is the y-intercept, is the formula for the slope-intercept form (the point where the line crosses the y-axis). Using y=mx+b to graph a line is typically simple. The standard form and the point-slope form are other formats for linear equations.Y equals mx plus b is actually the slope intercept form. Because it provides both a slope and an intercept, the form is known as a slope intercept form.

Given liner equation :

[tex]\frac{1}{3} z + y = 2[/tex]

[tex]y = 2 - \frac{1}{3} z[/tex]

[tex]y= - \frac{1}{3} z +2[/tex]

To learn more about slope intercept form refer,

https://brainly.com/question/1884491

#SPJ1

By rounding to 1 significant figure, estimate
40.73
×
1.06

Answers

By rounding to 1 significant figure, estimate answer is 40

What is significant figures?

Significant figures of a number in positional notation area unit digits within the number that area unit reliable and necessary to point the number of one thing.

Main  body:

1) 40.73 to 1 significant figure = 40.

2) 1.06 to 1 significant figure = 1.

3) Therefore, we just multiply 40 by 1, which equals 40.

FURTHER EXPLANATION (IF NEEDED):

With significant figures, whole numbers such as 10, 20, 50 etc. have 1 significant figure. This is because the 0 does not count as a significant figure.

With decimals, all numbers after the decimal point are also included as significant figures (even 0). E.g. 40.73 has 4 significant figures and 1.06 has 3 significant figures.

EXAMPLES (IF NEEDED):

Rounding to 2 significant figures:

40.73 = 42

1.06 = 1.1

Rounding to 3 significant figures:

40.73 = 40.7

1.06 = 1.06 (because it's already in 3 significant figures)

Rounding to 4 significant figures:

40.73 = 40.73 (already in 4 significant figures)

1.06 = 1.060 (0 is counted as a significant figure if there is a decimal or another number comes after that 0)

E.g. 1006 has 4 sig figs, while 1060 would have 3 sig figs because the last 0 is not counted as a significant figure. 10.60 would have 4 sig figs because of the decimal.

Hence , By rounding to 1 significant figure, estimate answer is 40.

to know more about significant figures , visit:

https://brainly.com/question/30169

#SPJ1

Help please !!!???!!!!

Answers

Answer:

the lenth of the court is 78 feet

a new washer and dryer set cost $1250 with an annual expenditure of $100 for electricity. The function C(n) =1250+100n/n represents the annual cost of operating the washer-dryer set as a function of n years of owning the set. Which of the following statement(s) about the function is true?

Answers

For the question "A new washer and dryer set costs $1250 with an annual expenditure of $100 for electricity..." None of the options are correct

What is annual expenditure?

The statement "The annual cost of the washer-dryer set will never exceed $100 during the life of the set" is false because the function C(n) is defined as C(n) = (1250 + 100(n))/n,

A washer-dryer set has an average lifetime of 10ten years

But given the function

C(n) = (1250 + 100(n))/n,

even after 5000 years, the washer-dryer set will still be above $100 as shown below

[tex]C(5000)=\frac{1250+100\left(500000\right)}{500000}[/tex]

C(5000)=100.0025

b) For the given function C(n) = (1250 + 100n) / n, the highest power of x is 1 in both the numerator and denominator. The leading coefficient of the numerator is 100 and the leading coefficient of the denominator is 1, so the horizontal asymptote is the line y = 100.

To find the vertical asymptotes of a rational function, we need to look for values of x that make the denominator equal to 0, but the numerator not equal to 0. These values of x will be the vertical asymptotes of the graph.

For the given function C(n) = (1250 + 100n) / n, the denominator is n, so the vertical asymptotes are x = 0.

Therefore, the asymptotes of the graph of C(n) = (1250 + 100n) / n are the horizontal asymptote y = 100 and the vertical asymptote x = 0.

The statement "The asymptotes of the function are at x = –12.5 and y = 100" is false because a function does not have asymptotes if it is defined for all values of x. In this case, the function C(n) is defined for all positive values of n, so it does not have asymptotes.

c)  The statement "The annual cost after 10 years is $225 assuming only purchase and electricity expenses" is false because the annual cost after 10 years is given by

C(10) = (1250 + 100*10)/10

C(10)=225

But

= 225 + 100 (10)

=225000

Therefore, the annual cost after 10 years is $225000, not $225.

Hence given this none of the Options are correct

Read more about annual expenditure

https://brainly.com/question/4611544

#SPJ1

CQ

A new washer and dryer set costs $1250 with an annual expenditure of $100 for electricity.

The function C(n) =1250+100n/n represents the annual cost of operating the washer-dryer set (besides the $100 for electricity) as a function of n years of owning the set.

Which of the following statement(s) about the function is true?

I. The annual cost of the washer-dryer set will never exceed $100 during the life of the set.

II. The asymptotes of the function are at x = –12.5 and y = 100.

III. The annual cost after 10 years is $225 assuming only purchase and electricity expenses.

F I only

G I and II only

H I and III only

J I, II, and III

a square room has a floor area of 64 square meters. the height of the room is 9 meters. what is the area of all 4 walls?

Answers

Answer:

288 m^2

Step-by-step explanation:

each side of the room is  8 m       ( 8x8 = 64)

So you have FOUR walls of widht 8 and height 9

   area =   FOUR * ( 9 *8) = 288 m^2

What is the probability that the students will have a pop quiz on a randomly selected day? a. 0.25 b. 0.52 c. 0.77 d. 0.90

Answers

The probability that the students will have a pop quiz on a randomly selected day will be A. 0.25.

How to calculate the probability?

Probability is the occurence of likely events. It is the area of mathematics that deals with numerical estimates of the likelihood that an event will occur or that a statement is true. An event's probability is a number between 0 and 1. In this case, 0 denotes the impossibility of the event and 1 represents certainty.

From the information, the probability that students will not have a quiz is 75%. The probability that the students will have a pop quiz on a randomly selected day will be:

= 1 - 75%

= 0.25

Learn more about probability on

brainly.com/question/24756209

#SPJ1

Complete question

The probability that students will not have a quiz is 75%. What is the probability that the students will have a pop quiz on a randomly selected day? a. 0.25 b. 0.52 c. 0.77 d. 0.90

Just answer it, dont have to show work, will mark brainly

Answers

The system of equations with the same solution set as the system of equations given is presented as follows:

5x + 3y = 10.x - 2y = -1.

How to obtain the system of equations with the same solution set?

The system of equations for this problem is presented as follows:

5x + 3y = 10.2x - 4y = -2.

Another system of equations will have the same solution set as the system given when it's equations are multiples of the equations in this problem.

This holds true for the first option, as:

The first equation is the same for both systems, 5x + 3y = 10.The second equation on the first option is x - 2y = -1, meaning that it was the second equation of the given system divided by 2.

More can be learned about a system of equations at https://brainly.com/question/24342899

#SPJ1

In a classroom 1/5 student get grade A; 3/10 got grade B and rest get grade C in math. Total number of students in the class is 50. Find how many students got grade C

Answers

Answer:

25

Step-by-step explanation:

make 1/5 and 3/10 with equivalent denominator which will give us 2/10 and 3/10 now add them and we get 5/10 5 is the half of ten so we will divide 50 over 2 and we will get 25

Other Questions
Consider the half reactions one more time. Will the reaction as written here occur spontaneously?Fe2+ (aq) +2 e- Fe(s) E = -0.447 VCu2+ (aq) +2 e- Cu(s) E = 0.342 VFe2+ (aq) + Cu(s) Cu+ (aq) + Fe(s)yes or no or need more information When one faces a situation with both attractive and unattractive characteristics, one is facing a(n) _____ conflict. The Lion and the MouseOnce when a Lion was asleep, a Mouse began running up and down upon him. This soon wakened the Lion, who placed his huge paw upon the Mouse, and opened his big jaws to swallow the little creature. Pardon, O Great Lion, cried the little Mouse, Forgive me this time, and I shall never forget it. Who knows? I may be able to do you a good turn one day. The Lion was so tickled at the idea of the Mouse being able to help him that he lifted his paw and let him go.Sometime later, the Lion was caught in a hunters trap. The hunters, who wanted to bring the Lion alive to the king, tied him to a tree while they went in search of a wagon to carry him. Just then, the Mouse happened to pass by. Seeing the sad plight of the Lion, the Mouse went up to him and soon gnawed away the ropes that bound the mighty animal. Was I not right? asked the little Mouse.1. List the characters in the fable.Start typing here...2. Summarize the plot.Start typing here...3. What moral lesson do you think this fable is trying to teach?Start typing here...4. Do you think this kind of storytelling is effective? Why or why not?Start typing here... Is this statement true or false? The cause and effect is the relationship between two events in a story. O true false sports teams travel together. they compete against other teams. this increases their a.diversity. b.norm generalization. c.role structure. d.cohesiveness. e.informal leadership. a chemical called endogenous pyrogens may be released from lymphocytes that are responding to a pathogen. what does this compound cause? multiple choice fever activation of nk cells antibody production inflammation Lim and Zebrack (2004) reported that for family caregivers of dementia, having social support is significantly associated with improved : Help please!!! a) what are the s and n axes on an s-n diagram, and what does the region to the left and below the plotted line indicate? A train is travelling along its railroad tracks. It slows down, goes around a turn, speeds back up, coasts at a constant velocity for awhile, then slows down again.How many times did the train accelerate?Responses1234A train is travelling along its railroad tracks. It slows down, goes around a turn, speeds back up, coasts at a constant velocity for awhile, then slows down again.How many times did the train accelerate?Responses1234 which of the following is the best example of shaping? responses a child receives five dollars each time he cleans his room. a child receives five dollars each time he cleans his room. an employee receives a termination notice after coming to work late every day over a period of three months. an employee receives a termination notice after coming to work late every day over a period of three months. a child gets candy from a dispenser one time but gets nothing from the dispenser the next two times. a child gets candy from a dispenser one time but gets nothing from the dispenser the next two times. a teacher rewards a student for sitting quietly for ten minutes on monday, fifteen minutes on tuesday, twenty minutes on wednesday, and thirty minutes on thursday. a teacher rewards a student for sitting quietly for ten minutes on monday, fifteen minutes on tuesday, twenty minutes on wednesday, and thirty minutes on thursday. a rat receives a mild shock each time it tries to open the door of its cage. Now drag the Low risk and High risk project points so their expected rates of return are 7% and 11%, respectively. If you could choose only one project to go forward, which would you choose? a. Project Low because its expected rate of return is close to its WACC. b. Project Average because its expected rate of return equals its WACC and the others are both below their respective WACC's. c. Project High because its expected rate of return is higher than for any of the other projects. d. Any are good choices because the WACC balances the risk. peripheral tolerance group of answer choices is caused by costimulation without tcr-mhc peptide interaction. is caused by tcr-mhc peptide interaction without costimulation. leads to anergy. both leads to anergy and is caused by costimulation without tcr-mhc peptide interaction. both leads to anergy and is caused by tcr-mhc peptide interaction without costimulation. 14. Which of these statements correctly describes the function y = 4x + 9?A. It is not a linear function, and its graph is not a straight line.B. It is a linear function, but its graph is not a straight line.C. It is not a linear function, but its graph is a straight line.D. It is a linear function, and its graph is a straight line. Complete the below table to calculate the price of a $1 million bond issue under each of the following independent assumptions (FV of $1, PV of $1, FVA of $1, PVA of $1, FVAD of $1 and PVAD of $1) (Use appropriate factor(s) from the tables provided. Enter your answers in whole dollars.):1.Maturity 10 years, interest paid annually, stated rate 10%, effective (market) rate 12%Amount Present ValueInterestPrinciplePrice of bond2.Maturity 10 years, interest paid semiannually, stated rate 10%, effective (market) rate 12%Amount Present ValueInterestPrinciplePrice of bond3.Maturity 10 years, interest paid semiannually, stated rate 12%, effective (market) rate 10%Amount Present ValueInterestPrinciplePrice of bond4.Maturity 20 years, interest paid semiannually, stated rate 12%, effective (market) rate 10%Amount Present ValueInterestPrinciplePrice of bond5.Maturity 20 years, interest paid semiannually, stated rate 12%, effective (market) rate 12%A mount Present ValueInterestPrinciplePrice of bond which sentence is punctuated correctly? question 4 options: a) chris researched, visited, and booked the location, zach hired and supervised the caterers, and stephanie ordered and sent the invitations. b) chris researched, visited, and booked the location; zach hired and supervised the caterers; and stephanie ordered and sent the invitations. 2._____________This mountain range forms a natural boundary between France and Spain. The left and right cervical plexuses are located deep on each side of the neck, immediately lateral to cervical vertebrae ______. presented below is selected information for sheridan company. answer the questions asked about each of the factual situations. (do not leave any answer field blank. enter 0 for amounts.) 1. sheridan purchased a patent from vania co. for $1,160,000 on january 1, 2018. the patent is being amortized over its remaining legal life of 10 years, expiring on january 1, 2028. during 2020, sheridan determined that the economic benefits of the patent would not last longer than 6 years from the date of acquisition. what amount should be reported in the balance sheet for the patent, net of accumulated amortization, at december 31, 2020? Using the same country you chose for the mapping task as your subject, research important demographic factors using the World Factbook. Create a country profile, making sure to include the following information:gross domestic product per capitalife expectancyliteracy rateinfant mortality ratemajor agricultural productsmajor industriesmain exports and importsUsing this information, write a short profile of the country you chose. AUSTRAILA two different two-digit whole numbers are selected at random. what is the probability that their product is less than 200. express your answer as a common fraction. (hints: (l) there are 90 different two-digit numbers, (2) the pair {10, 11} produces the smallest product and the pair {11, 18} produces the largest product less than 200).